Pagina 33 di 33

Problema 119.

Inviato: 03 ago 2012, 17:17
da jordan
Problema 119 da qui:
jordan ha scritto:Trovare tutti gli interi positivi $n$ tali che l'insieme $\{n,n+1,n+2,\ldots,n+9\}$ può essere partizionato in due sottoinsiemi a prodotto costante.
<enigma> ha scritto:Se $11$ divide uno fra quei dieci numeri abbiamo un assurdo perché non può dividerne un altro ma il prodotto degli elementi di ciascuno dei due sottoinsiemi ha il fattore $11$. Siano $S_1$ e $S_2$ i due insiemi e $P$ il prodotto degli elementi di ciascuno: allora $\prod _{k=0}^9 (n+k) \equiv -1 \pmod {11}$, ma anche $=\prod_{i \in S_1} i \prod_{j \in S_2} j \equiv P^2$; tuttavia $P^2 \equiv -1 \pmod {11}$ è assurdo. Tale partizione è quindi impossibile.

Problema 120.

Inviato: 03 ago 2012, 17:19
da jordan
Problema 120 da qui:
<enigma> ha scritto:Risolvere negli interi positivi $(2^a-1)(3^b-1)=c!$.
[Mathematical Reflections]
Soluzione postata qui (in basso c'è direttamente il pdf)

121

Inviato: 22 ott 2012, 13:47
da jordan
Problema 121 da qui
jordan ha scritto:121. (Own) notazioni: Siano $\text{gpf}(x)$ e $\text{lpf}(x)$ il piu' grande fattore primo e il piu' piccolo fattore primo che dividono un intero $x>1$ e $\upsilon_p(x)$ la valutazione p-adica di $x$.
Definiamo infine la funzione aritmetica $f(\cdot):\mathbb{N}_0\to \mathbb{N}_0$ di modo tale che :
$f(n)=1$ se $n=1$ oppure se $\text{gpf}(n)<100$
$f(n)=\text{lpf}\left(n\prod_{p\in \mathbb{P}\cap[1,100]}{p^{-\upsilon_p(n)}}\right)$ altrimenti.

In altre parole, f(n) e' il piu' piccolo primo che divide n, che è maggiore di 100; se tale primo non esiste allora f(n)=1.


Dimostrare che, fissata una costante $C>10^{40}$, esistono infiniti interi positivi $n$ tali che $C<f(n)<f(n+1)<f(n+2)<...<f(n+99)<C^2$.
Leonida ha scritto:Lemma: esistono almeno 100 numeri primi $p$ tali che $C < p < C^2$.
Dim.: dimostro per induzione su $k$ che tra $C$ e $2^k C$ vi sono almeno $k$ numeri primi.
Passo base: $k=1$. Segue dal il Teorema di Chebyshef (o Postulato di Bertrand), il quale afferma che esiste almeno un primo tra $C$ e $2C$, purchè $C > 1$.
Passo induttivo: supponiamo esistano $k$ numeri primi tra tra $C$ e $2^k C$. Ancora per il Teorema di Chebyshef, esiste almeno un primo tra $2^k C$ e $2^{k+1} C$, perciò tra $C$ e $2^{k+1} C$ vi sono almeno $k+1$ primi e il passo induttivo è verificato.
In particolare esistono almeno 100 numeri primi $p$ tali che $C < p < 2^{100} C$. Ma $2^{100} C < 8^{34} C < 10^{34} C < 10^{40} C < C^2$ è vera poichè per ipotesi $C > 10^{40}$, da cui segue il Lemma.
Indico con $101 < 103 < ... < p_{C-1} \leq C$ i primi compresi tra 100 e $C$ e con $C < p_{C}< p_{C+1}< ... < p_{C+99} < C^2$ cento numeri primi compresi tra $C$ e $C^2$ (esistono per il Lemma). E' sufficiente scegliere $n$ in modo che risolva il seguente sistema di congruenze:
\[
\left\{
\begin{array}{l}
n \equiv 1 \pmod {101} \\
n \equiv 1 \pmod {103} \\
n \equiv 1 \pmod {...} \\
n \equiv 1 \pmod {p_{C-1}} \\
n +i \equiv 0 \pmod {p_{C +i}}, \forall i, 0 \leq i \leq 99
\end{array}
\right.
\]
Innanzitutto per il Teorema Cinese del Resto esistono infinite soluzioni a questo sistema di congruenze, dato che i moduli sono a due a due coprimi. Inoltre, se $101 \leq p \leq p_{C-1}$, $n +i \equiv i+1 = 1, ... , 100 \not \equiv 0 \pmod p$.
Pertanto $f(n +i) \geq p_{C} > C$, $\forall i, 0 \leq i \leq 99$.
Allora $f(n +i) = p_{C +i}$ ; di conseguenza $f(n) < f(n+1) < ... < f(n +99) < C^2$ è vera perchè lo è $p_C < p_{C +1} < ... < p_{C +99} < C^2$. In conclusione, si ha $C < f(n) < f(n+1) < ... < f(n +99) < C^2$ che è la tesi.
P.S.: dato che questa soluzione è un po' barata, se qualcuno ne ha una che non passi per il Postulato di Bertrand mi sembra giusto cedergli il testimone della staffetta.

122.

Inviato: 22 ott 2012, 13:48
da jordan
Problema 122 da qui:
jordan ha scritto:Dal 4 agosto Leonida non è si è fatto piu' vivo sul forum, e nessuno ha voluto postare quello nuovo, per cui..

Siano a e b due interi positivi fissati. Mostrare che esiste solo un numero finito di interi positivi n tali che $\displaystyle \left(a+\frac{1}{2}\right)^n+\left(b+\frac{1}{2}\right)^n$ è intero.
Ido Bovski ha scritto:$\displaystyle \left(a+\frac{1}{2}\right)^n+\left(b+\frac{1}{2}\right)^n=\frac{(2a+1)^n+(2b+1)^n}{2^n}=\frac{1}{2^n}\sum_{k=0}^n 2^k\binom{n}{k}(a^k+b^k)$
  • Se $n\neq 0$ è pari, allora $2^n\nmid (2a+1)^n+(2b+1)^n$, infatti
    $\displaystyle \sum_{k=0}^n 2^k\binom{n}{k}(a^k+b^k)\equiv \sum_{k=0}^1 2^k\binom{n}{k}(a^k+b^k)\equiv 2+2n(a+b)\equiv 2 \not\equiv 0 \pmod 4$
    Dunque non esiste un $n$ che soddisfa le condizioni richieste.
  • Se $n$ è dispari, allora, poiché $2|(2a+1)+(2b+1)$, per LTE
    $\nu_2((2a+1)^n+(2b+1)^n)=\nu_2((2a+1)+(2b+1))=1+\nu_2(a+b+1)$
    Pertanto $2^n|(2a+1)^n+(2b+1)^n$ per $ \lceil (1+\nu_2(a+b+1))/2 \rceil $ scelte di $n$.

123

Inviato: 22 ott 2012, 13:49
da jordan
Problema 123 da qui:
Ido Bovski ha scritto:Sia $p$ un numero primo. Dato un $a\in\mathbb{Z}$ tale che $\gcd(a, p!)=1$, dimostrare che $p!|a^{(p-1)!}-1$.
Se a $p$ primo si sostituisce un $n\in\mathbb{N}$ libero da quadrati, l'enunciato è ancora vero?
jordan ha scritto:Dai due interi $a,b$ possiamo affermare che $a \mid b$ se e solo se $\upsilon_q(a)\le \upsilon_q(b)$ per ogni $q \in \mathbb{P}$.
Adesso e' necessario (e sufficiente) mostrare tale disuguaglianza per tutti i primi $q\le p$: fissato un tale $q$ infatti abbiamo in particolare che $q\mid p! \implies \text{gcd}(a,q)=1$ per cui $\displaystyle \upsilon_q(a^{(p-1)!}-1)=$ $\displaystyle \upsilon_q((a^{q-1})^{(p-1)!/(q-1)}-1)=$ $ \displaystyle \upsilon_q(a^{q-1}-1)+\upsilon_q(\frac{(p-1)!}{q-1})$ $=\displaystyle \upsilon_q(a^{q-1}-1)+\upsilon_q((p-1)!)$ (per LTE e T.di Fermat). Affinche' $\upsilon_q(p!) \le \upsilon_q(a^{(p-1)!}-1)$ dobbiamo quindi mostrare in modo equivalente che $\upsilon_q(p!) \le \upsilon_q(a^{q-1}-1)+\upsilon_q((p-1)!)$ che e' vera se e solo se $\upsilon_q(p) \le \upsilon_q(a^{q-1}-1)$ per ogni primo $q \le p$ e intero $a$ coprimo con $p!$.

E' ovvio adesso che, se $\mu(p)\neq 0$ allora la divisibilità e' sempre verificata, difatti $\displaystyle \upsilon_q(p) \in \{0,1\}$ e $\upsilon_q(a^{q-1}-1) \ge 1$. []

Re: Staffetta tdn

Inviato: 22 ott 2012, 13:50
da jordan
Problema 124 da qui:
jordan ha scritto:Sia dato un intero $ a \ge 3 $. Mostrare che esiste un intero $ n $ con esattamente 2012 divisori primi tale che $ n\mid (a^n+1) $
nobu ha scritto:Voglio dimostrare che dati 2 interi $a\geq 3$ e $k\geq 1$, esiste un intero $n$ con esattamente $k$ divisori primi t.c. $n\mid a^n+1$.

Distinguo 2 casi:
  • Se $a+1$ non è una potenza di 2, dimostro per induzione su $k$:
    PASSO BASE: $k=1$. Scelgo $n=q$ con $q$ primo dispari che divide $a+1$, allora $q\mid a+1\mid a^q+1$.
    PASSO INDUTTIVO: $k\to k+1$. Per ipotesi induttiva ho $n$ intero con esattamente $k$ divisori primi t.c. $n\mid a^n+1$.
    Considero un primo dispari $q$ che divide $n$ (che esiste perchè all'inizio ho preso un primo dispari che divide $a+1$), allora ho che $nq^\alpha \mid a^{nq^\alpha}+1$ per ogni $\alpha$ intero positivo, dato che $n\mid a^n+1\mid a^{nq^\alpha}+1$ e per LTE vale $\upsilon_q(a^{nq^\alpha}+1)=\upsilon_q(a^n+1)+\alpha$.
    Voglio dimostrare quindi che esiste un primo $p$ tale che $p\mid a^{pnq^\alpha}+1$, ma $p\nmid nq^\alpha$; avrei quindi che $pnq^\alpha$ ha esattamente $k+1$ divisori primi e $pnq^\alpha\mid a^{pnq^\alpha}+1$.
    Ma $p\mid a^{pnq^\alpha}+1 \iff p\mid a^{nq^\alpha}+1$, quindi quello che voglio equivale a dimostrare che esiste $\alpha$ t.c. $a^{nq^\alpha}+1$ ha dei divisori primi che non dividono $n$, ma ciò è facilmente verificabile perchè aumentando $\alpha$ la valutazione $p$-adica dei primi che dividono $n$ rimane la stessa tranne che per $q$, ma quindi $a^{nq^\alpha}+1$ diventa "troppo grande" per avere solo primi che dividono anche $n$.
  • Se $a+1$ è una potenza di 2 c'è qualcosa da sistemare nel passo base, perchè non posso scegliere un primo $q$ dispari t.c. $q\mid a+1$.
    Quindi se $k=1$ scelgo $q=2$ che divide $a+1$ e ho quindi che $2\mid a^2+1$.
    Se $k=2$ voglio quindi dimostrare che esiste un primo dispari che divide $a^2+1$, ma questo è vero perchè $a^2+1\equiv 2 \pmod 4$.
    Tutto il resto è uguale a prima.

125

Inviato: 22 ott 2012, 13:53
da jordan
Problema 125 da qui:
nobu ha scritto:Sia $a_0,a_1,a_2,...$ una successione di numeri interi positivi tali che $(a_{i+2},a_{i+1})>a_i$ per ogni $i\geq0$.
Dimostrare che $a_n\geq 2^n$ per ogni $n\geq 0$.
jordan ha scritto:Sono bboypa su Mathlinks, e ricordavo di aver risolto questo problema tempo addietro, difatti:
bboypa ha scritto:
April ha scritto:Let $ a_0$, $ a_1$, $ a_2$, $ \ldots$ be a sequence of positive integers such that the greatest common divisor of any two consecutive terms is greater than the preceding term; in symbols, $ \gcd (a_i, a_{i + 1}) > a_{i - 1}$. Prove that $ a_n\ge 2^n$ for all $ n\ge 0$.
Our aim $ a_n \ge 2^n$(*) is true for $ n \in \{0,1\}$, in fact $ a_1=1$ would imply $ 1= (a_2,a_1)>a_0 \in \mathbb{N}_0$, absurd. It is also true that $ \{a_n\}_{n \in \mathbb{N}}$ is strictly incrasing since $ \min\{a_{n+1},a_{n+2}\}$ $ \ge (a_{n+1},a_{n+2})$ $ >a_n$. Now if (*) is true for $ n \in \{0,1,\ldots,n-1\}$ then it is true also for $ n$: in fact we have $ a_n-a_{n-1}$ $ \ge (a_n-a_{n-1},a_{n-1})$ $ =(a_n,a_{n-1}) >a_{n-2} \implies$ $ a_n > a_{n-1}+a_{n-2}$. Now if $ \frac{a_{n-1}}{3} \ge (a_{n-1},a_n)> a_{n-2}$ then we are done since $ a_n>a_{n-1}+a_{n-2} \ge 4a_{n-2} \ge 2^n$. Otherwise $ \frac{a_{n-1}}{2}=(a_n,a_{n-1})$. Now if $ a_n \ge 2a_{n-1}$ we are done, otherwise it means that $ 2 \mid a_{n-1}$, $ 3 \nmid a_{n-1}$ and $ a_n=\frac{3}{2}a_{n-1}$. Now if $ \frac{a_{n-1}}{4} \ge a_{n-2} \implies a_n>a_{n-1} \ge 2^n$, in the last last case (since $ 3 \nmid a_{n-1}$) we must have $ (a_{n-2},a_{n-1})=\frac{a_{n-1}}{2}$, but $ \frac{a_{n-1}}{2}=(a_n,a_{n-1})>a_{n-2}=\frac{a_{n-1}}{2}$, contradiction.
Original thread: qui (c'è anche una soluzione che usa un'altra strada se non ricordo male)
Proposed by Morteza Saghafian, Iran, IMO Shortlist 2008

126

Inviato: 22 ott 2012, 13:53
da jordan
Problema 126 da qui:
jordan ha scritto:Dimostrare che per ogni coppia di interi positivi $m,n$ vale: $m!n!(m+n)!$ divide $(2n)!(2m)!$

Ps. IMO 1972 O.o
bĕlcōlŏn ha scritto:$ \textbf{Lemma} $:
$ \lfloor a \rfloor + \lfloor b \rfloor + \lfloor a+b \rfloor \leq \lfloor 2a \rfloor + \lfloor 2b \rfloor $

Per la dimostrazione considero $ a = \lfloor a \rfloor + \epsilon_1 $ con $ \epsilon_1 < 1 $, e $ b=\lfloor b \rfloor + \epsilon_2 $ con $ \epsilon_2 < 1 $. Ci sono due casi:
-Caso 1: $ \epsilon_1+\epsilon_2 < 1 $. In tal caso $ LHS = 2(\lfloor a \rfloor + \lfloor b \rfloor) \leq \lfloor 2a \rfloor + \lfloor 2b \rfloor $, poiché $2\lfloor a \rfloor \leq \lfloor 2a \rfloor$ abbastanza chiaramente.
-Caso 2: $ 1\leq \epsilon_1+\epsilon_2 < 2 $. In tal caso sicuramente almeno uno fra $ \epsilon_1,\epsilon_2 $ deve essere $ \geq 0.5 $. Il che vuol dire che sicuramente $ LHS=2\lfloor a \rfloor + 2\lfloor b \rfloor + 1 $ mentre nell'RHS sicuramente almeno uno (wlog) $ \lfloor 2a \rfloor $ è $ 2\lfloor a \rfloor + 1 $ e invece $ \geq \lfloor 2b \rfloor \geq 2\lfloor b \rfloor $. Quindi $LHS \leq RHS$.

A questo punto per concludere basta applicare questa http://it.wikipedia.org/wiki/Identit%C3 ... e_Polignac chiamando $ \dfrac{m}{p^k} = a, \dfrac{n}{p^k} = b $ e applicando per ogni esponente $k$ la disuguaglianza di sopra. In questo modo $v_p(m!n!(m+n)!) \leq v_p((2m)!(2n)!)$ per ogni $p$ e quindi si ha la divisibilità.

127

Inviato: 22 ott 2012, 13:54
da jordan
Problema 127 da qui:
bĕlcōlŏn ha scritto:Trova tutti i numeri naturali $n$ per i quali esiste una permutazione $(p_1,p_2,\dots,p_n)$ di numeri $(1,2,\dots,n)$ tale che gli insiemi formati dai numeri $p_i+i \quad \forall 1\leq i \leq n$ e $p_i-i \quad \forall 1\leq i \leq n$ sono sistemi completi di residui modulo $n$.
jordan ha scritto:Definiamo $\sigma_1(i):=p_i+i$ and $\sigma_2(i):=p_i-i$ le due permutazioni di $i$, per ogni $i=1,2,\ldots,n$.

Dal momento che $\{\sigma_1(1),\sigma_1(2),\ldots,\sigma_1(n)\}$, $\{\sigma_2(1),\sigma_2(2),\ldots,\sigma_2(n)\}$ e $\{p_1,p_2,\ldots,p_n\}$ sono tre permutazioni di $\{1,2,\ldots,n\}$ allora $\displaystyle \sum_{i=1}^n{\sigma_1^2(i)+\sigma_2^2(i)} \equiv \sum_{i=1}^n{i^2+i^2}\pmod n$, cioè $\displaystyle 2\sum_{i=1}^n{p_i^2+i^2}\equiv \sum_{i=1}^n{i^2+i^2} \pmod n \implies n \mid 2\sum_{i=1}^n{p_i^2} \equiv \frac{1}{3}n(n+1)(2n+1)$: questo significa che $n$ non è multiplo di $3$.

Dal momento che $\{\sigma_2(1),\sigma_2(2),\ldots,\sigma_2(n)\}$ e' una permutazione di $\{1,2,\ldots,n\}$ allora $\displaystyle \sum_{i=1}^n{\sigma_2(i)} \equiv \sum_{i=1}^n{i} \pmod n$, cioè $n \mid \displaystyle \sum_{i=1}^n{p_i} \equiv \frac{1}{2}n(n+1)$: questo significa che $n$ deve essere dispari.

Assumiamo ora che $\text{gcd}(n,6)=1$ e poniamo $p_i\equiv 2i \pmod n$ per $i=1,2,\ldots,n$ (e' difatti una permutazione visto che $n$ e' dispari). Adesso $\sigma_1(i) \equiv 3i \pmod n$ (e' ancora una permutazione visto che $n$ non è multiplo di $3$) e banalmente $\sigma_2(i)= i$. []

Ps. Bell'esercizio

128

Inviato: 22 ott 2012, 13:55
da jordan
Problema 128 da qui:
jordan ha scritto:Sia $f(x)$ un polinomio non costante a coefficienti interi. Sia $S(f(x))$ l'insieme di tutti e soli i primi $p$ tali che esiste $n$ intero con $p\mid f(n)$. Mostrare che $S(f(n))$ non e' finito.

(Schur)
Ido Bovski ha scritto:Sia $\displaystyle f(x)=\sum_{i=0}^m a_ix^i$. Se $a_0=0$, chiaramente $p\mid f(p)$ per ogni $p$ primo. Analizziamo ora il caso in cui $a_0\neq 0$.
L'equazione $f(x)=\pm 1$ ha un numero finito di soluzioni, pertanto $S(f(x))$ non è vuoto. Supponiamo che $S(f(x))$ sia finito e chiamiamo $p_1, p_2,\ldots, p_k$ i suoi elementi. Allora, detto $P=p_1p_2\ldots p_k$, abbiamo che $\displaystyle f(a_0Px)=a_0 \left(1+\sum_{i=1}^m a_ia_0^{i-1}P^ix^i \right)=a_0g(x)$. Per lo stesso ragionamento di prima, esistono $n$ intero e $q$ primo tali che $q\mid g(n)$. Allora $q\mid f(a_0Pn)$, ma ciò è assurdo poiché $q\nmid P$. Dunque, $S(f(x))$ non è finito.

Re: Staffetta tdn

Inviato: 22 ott 2012, 13:56
da jordan
Problema 129 da qui:
Ido Bovski ha scritto:Determinare tutte le soluzioni intere dell'equazione
$$\frac{x^7-1}{x-1}=y^5-1.$$
jordan ha scritto:\[ y^5-1=\frac{x^7-1}{x-1}=1+x(x+1)(x^2+x+1)(x^2-x+1) \]
Dato che $2\mid x(x+1)$ allora
\[ y^5-1\equiv \frac{x^7-1}{x-1} \equiv 1 \pmod 2\]
Abbiamo che $\displaystyle q(x):=\frac{x^7-1}{x-1}$ e' il settimo polinomio ciclotomico con dominio $\mathbb{R}\setminus\{1\}$; ora $q(0)=q(-1)=1$ e $q(2)=2^7-1$, che non sono esprimibili nella forma $y^5-1$; in tutti gli altri casi abbiamo $|x-1|\ge 2$, per cui possiamo definire un primo $p$ tale che $p \mid x-1$.
$\bullet$ Caso 1: $7\nmid x-1$.
In tal caso $\displaystyle \upsilon_p\left(\frac{x^7-1}{x-1}\right)=\upsilon_p(7)=0$, in altre parole se $p\mid x-1$ allora $p\nmid \displaystyle \frac{x^7-1}{x-1}$. Allora $7\mid \text{ord}_p(x) \mid \varphi(p)=p-1$, cioè tutti i primi divisori di $x^7-1$ (e quindi anche di $\displaystyle \frac{x^7-1}{x-1}$) hanno resto $1$ modulo $7$. Abbiamo quindi
\[ \frac{x^7-1}{x-1} \equiv 1 \pmod 7 \implies y^5-1 \equiv 1 \pmod 7 \implies y \equiv 4\pmod 7 \]
Notiamo infine che $y-1 \mid y^5-1 = \frac{x^7-1}{x-1} \mid x^7-1$, per cui anche tutti i divisori primi di $y-1$ sono $\equiv 1 \pmod 7$, i.e. $y-1 \equiv 1 \pmod 7$, assurdo.
$\bullet$ Caso 2: $7\mid x-1$.
Ragionando come sopra, la fattorizzazione canonica di $\displaystyle \frac{x^7-1}{x-1}$ sarà $7 \cdot p_1^{\alpha_1} \cdot p_2^{\alpha_2} \cdot \ldots \cdot p_k^{\alpha_k}$ dove $p_i\equiv 1 \pmod 7$ per ogni $i=1,2,\ldots,k$; in particolare
\[ \frac{x^7-1}{7(x-1)} \equiv 1 \pmod 7 \]
Dato che $7 \mid \frac{x^7-1}{x-1}$ allora anche $7\mid y^5-1$, ma $\text{gcd}(5,\varphi(7))=1$, per cui anche $7\mid y-1$ (e, vista la fattorizzazione sopra, $7 || y-1$). Esisteranno quindi degli interi $X,Y$ tali che $x=7X+1$ e $y=7Y+1$ (con $7\nmid Y$ e $X\neq 0$). Abbiamo:
\[ \sum_{i=0}^6{(7X+1)^i}=(7Y+1)^5-1 \]
che in $\mathbb{Z}/7^2\mathbb{Z}$ implica
\[ \sum_{i=0}^6{(7iX+1)}=5\cdot 7 Y \implies 7+7X\sum_{i=0}^6{i}=5\cdot 7 Y \implies 7=5\cdot 7 Y\]
cioè $5Y \equiv 1 \pmod 7$, i.e. $Y \equiv 3 \pmod 7$.
D'altra parte sappiamo che $\frac{x^7-1}{7(x-1)} \equiv 1 \pmod 7$ per cui anche $\frac{1}{7}(y^5-1) \equiv 1 \pmod 7 \implies Y=\frac{1}{7}(y-1) \equiv 1 \pmod 7$, assurdo. []

130

Inviato: 22 ott 2012, 13:57
da jordan
Problema 130 da qui:
jordan ha scritto:Trovare tutti gli interi positivi $n$ tali che $3^n+4^n \mid 5^n$.
kalu ha scritto:Se $ 4 \mid n $ per Fermat $ 3^n+4^n \equiv 2 \pmod{5} $, assurdo.
Se $ n $ è dispari $ 7 \mid 3^n+4^n \to 7 \mid 5^n $, assurdo.
Quindi $ n $ è della forma $ 2k $, con $ k $ dispari. Allora deve valere $$9^k+16^k=5^z$$ per qualche $ z\leq 2k $.
Suppongo che esista un primo $ q $ tale che $ q \mid k $. Applico allora il lemma LTE: $$\displaystyle v_5\biggl(\frac{9^k+16^k}{9^{\frac{k}{q}}+16^{\frac{k}{q}}}\biggl)=v_5({9^k+16^k})-v_5(9^{\frac{k}{q}}+16^{\frac{k}{q}})=v_5(q)\leq 1$$ Dato che banalmente $ \displaystyle \frac{9^k+16^k}{9^{\frac{k}{q}}+16^{\frac{k}{q}}}>1 $, Deve valere $ q=5 $, e $ \displaystyle \frac{9^k+16^k}{9^{\frac{k}{5}}+16^{\frac{k}{5}}}=5 $.
Noto che: $$\displaystyle\sqrt[5]{\frac{5(9^{\frac{k}{5}}+16^{\frac{k}{5}})}{2}}=\sqrt[5]{\frac{9^k+16^k}{2}}\geq \frac{9^{\frac{k}{5}}+16^{\frac{k}{5}}}{2} \to \frac{9^{\frac{k}{5}}+16^{\frac{k}{5}}}{2} \leq \sqrt[4]{5}$$
Chiaramente assurdo.
Quindi non esistono primi divisori di $ k $: allora $ k=1 $, che in effetti soddisfa.
Concludo che l'unica soluzione è $ n=2 $.

131

Inviato: 22 ott 2012, 13:58
da jordan
Problema 131 da qui:
kalu ha scritto:Qualcosa di generale e (credo) utile da risolvere e tenere bene a mente:

a) "Own". Trovare tutte le quintuple $ (a, b, n, p, z) $ di interi positivi, con gcd$ (a, b)=1 $, max$ \{a, b\}>1 $, $ n>1 $ dispari e $ p $ primo, tali che: $$a^n+b^n=p^z$$

b) "Own". Siano $ a, b, n, p, z $ interi positivi con gcd$ (a, b)=1 $, $ n>1 $, $ p $ primo, tali che: $$a^n-b^n=p^z$$ Dimostrare che $ n $ è primo.
Testo nascosto:
In un po' di thread abbastanza recenti potreste trovare più di un hint :!:
Forse basterà risolverne uno per avere il testimone :|
NB Gli "own" sono fra virgolette perchè si tratta di equazioni talmente generali che non si può credere che nessuno ci abbia pensato prima di me xD
jordan ha scritto: $\bullet$ Se $p=2$ siano $\alpha, \beta, A, B$ univocamente determinati tali che $a=2^A\alpha, b=2^B\beta$ e wlog $A\le B$ con $2\nmid \alpha\beta$; abbiamo
\[ 2^z=a^n+b^n=2^{nA}\left(\alpha^n+2^{n(B-A)}\beta^n\right) \]
Se $A\neq B$ allora $\alpha^n+2^{n(B-A)}\beta^n$ e' dispari, maggiore di $1$, e divide $2^z$, assurdo. Resta il caso $A=B$, allora $2^{z-nA}=\alpha^n+\beta^n$ e $z-nA=\upsilon_2(\alpha^n+\beta^n)=\upsilon_2(\alpha+\beta) \le \text{log}_2(\alpha+\beta)$. Per cui:
\[ \alpha^3+\beta^3\le \alpha^n+\beta^n=2^{z-nA}\le 2^{\text{log}_2(\alpha^2-\beta^2)-1} \le \alpha+\beta \implies \alpha=\beta=1 \]
Per cui esistono solo le soluzioni banali $(a,b,n,z)=(2^k,2^k,n,kn+1)$.

$\bullet$ Se $p\ge 3$, dato che $3\le a+b \mid a^n+b^n = p^z$, allora $a+b, a^n+b^n$ sono potenze di $p$. Per LTE:
\[ z=\upsilon_p(p^z)=\upsilon_p(a^n+b^n)=\upsilon_p(a+b)+\upsilon_p(n) \implies a^n+b^n=p^z \le p^{\text{log}_p(a+b)+\text{log}_p(n)}=n(a+b) \]
Per le medie $n(a+b)\ge a^n+b^n > \frac{1}{2^{n-1}}(a+b)^n \implies a+b < 2n^{\frac{1}{n-1}}\le 2\sqrt{3} \implies a+b \le 3$ cioè wlog $a=1, b=2$. Dobbiamo quindi risolvere
\[ 2^n+1=p^z \]
per qualche $n$ dispari $>1$, ma $3\mid 2^n+1^n \implies p=3$. Modulo $4$ abbiamo $z$ pari, per cui $2^n=(3^{\frac{z}{2}}+1)(3^{\frac{z}{2}}-1)$,entrambi i fattori sono potenze di $2$ e differiscono di $2$, per cui devono essere $2$ e $4$ da cui $z=2$. Da cui l'unica soluzione $2^3+1^3=3^2$. []